Kayden invested $6,300 in an account in the year 2006, and the value has been growing exponentially at a constant rate. The value of the account reached $8,200 in the year 2010. Determine the value of the account, to the nearest dollar, in the year 2019.

Answers

Answer 1

The answer to the expositional growing account will be $14838.

What is exponential function?

The mathematical formula for an exponential function is f (x) = ax, where x is a variable and a constant known as the base of the function, which must be greater than zero. The transcendental number, or e, which is roughly equivalent to 2.71828, is the most frequently used exponential function base.

Let suppose in the xth year, the value of the money is 6300* [tex]a^{4}[/tex]

⇒8200 = 6300* [tex]a^{4}[/tex]

⇒a = 1.068

So 6300 *[tex]a^{13}[/tex]

= $14838

Hence the value of the account will be $14838.

Learn more about expositional, by the following link

https://brainly.com/question/2456547

#SPJ1


Related Questions

Find the geometric mean between each pair of numbers. 8 and 12

Answers

The Geometric Mean between the pair of numbers 8 and 12 is 4√6   .

Geometric Mean between the numbers x and y is calculated using the formula ,

GM = √(x * y),

where GM represents the Geometric Mean

In the question ,

it is given that ,

the pair of numbers are 8 and 12 .

we need to find the geometric mean between them ,

By using the Geometric Mean formula from above ,

we get ,

GM = √(8 × 12)

GM = √96

GM = 4√6

Therefore , The Geometric Mean between the pair of numbers 8 and 12 is   4√6  .

Learn more about Geometric Mean here

https://brainly.com/question/3856884

#SPJ9

Find the solution to the linear equation.
9x8=9x+6

Answers

Multiply the numbers

9 x 8 = 72

Subtract 6 from both sides

72 - 6 = 9x + 6-6

Simplify

66 = 9x

Divide both sides by the same factor

66 = 9x

66/9 = 9x/9

Simplify

X = 22/3

Answer: x = 22/3

Sorry if I didn’t do it right

Answer:

72=9x+6

x= 7.111111111111111111111111111111111111111111111111111111111111and on

Step-by-step explanation:

In the diagram, the lines L and M appear to be parallel. How do you know for sure?​

Answers

Proved  that the line L and M are parallel lines

Here we have to prove that L and M are parallel

One angle at the point at F is given that 139 degrees

We know sum of angles on a straight line is 180 degrees

Then the other angle will be

139 + a = 180

a = 180 - 139

a = 41 degrees

Now consider the triangle and find the value of x

The sum of interior angles of a triangle is 180 degrees

then

(3x+8) + (3x) + (9x+7) = 180

3x+8 +3x +9x + 7 = 180

15x + 15 = 180

15x = 180 - 15

15x = 165

x = 165/15

x  = 11

Then the value of angle 3x +  8 = 3 × 11 + 8

= 33 + 8

= 41 degrees

Before that we have solved the angle at f = 41 degrees, therefore both are equal angles  which is called corresponding angles

Therefore, the line L and M are parallel

Hence, proved  that the line L and M are parallel lines

Learn more about parallel lines here

brainly.com/question/12274210

#SPJ9

What is the degree of the polynomial below?
x +32x² + 4x² - 4x
A.
7
B.
6
C.
5
D.
4

Answers

Answer:

2

Step-by-step explanation:

Degree is the largest exponent        = 2   (as written)

A​ salesperson's commission rate is ​4%. What is the commission from the sale of %31,000 worth of​ furnaces

Answers

The commission on the sale of 31000 worth of furances when the rate of commission is 4% is given by 1240.

Given that, a salesperson's commission rate is 4%.

Sale of person is 31000.

So the commission of the that sales is = 31000*4% = 31000*(4/100) = 310*4 = 1240

Hence the commission of the sale is 1240.

To know more about Sale refer to:

https://brainly.com/question/78672

#SPJ9

Please help - giving extra points

4) The Opus Supreme Opera House has 300 seats. 90 of the seats are in the balcony.
What percent of the seats are in the balcony?
Pick the model that represents the problem.
0%
0%
90
%
?
90
What percent of the seats are in the balcony?
()
100%
300
100%
300

Answers

Answer: 30%; Second Model

Step-by-step explanation:

Since it is 90/300, simplified into 3/10

The percentage is 30% and the second model shows 3/10, the first one shows 2/5.

Part B
What is the ratio of the unknown amount (x) invested in company A to the known amount ($24) invested in company B? Write the answer as a
fraction.

Answers

The ratio of the amounts is x/24.

The amount invested in company A is unknown. The amount is represented by the variable "x". The amount invested in company B is $24. We need to find out the ratio of the amount invested in company A to that of company B. We need the answer as a fraction.

Let the ratio be represented by the variable "r". A ratio compares two numbers. A ratio expresses how much of one thing there is in comparison to another. A fraction is a portion of a whole or, more broadly, any number of equal pieces. In ordinary English, a fraction denotes the number of components of a specific size.

r = x/24

Hence, the ratio of the amount invested in company A to that of company B is x/24.

To learn more about ratio, visit :

https://brainly.com/question/13419413

#SPJ1

HELPPPPPPPP!!! Picture included

Answers

The equation that could be used to find the measure of angle B is Sin B = √5/5.

According to the question,

We have the following information:

We have a right-angled triangle with all its sides given.

Now, in order to find the measurement of angle B, we will use trigonometric functions. Two functions Sin and Cos are given in options. So, we will find the values of angle B using these two.

Cos B = AB/BC

Cos B = 6/3√5

Cos B = 2/√5

Now, multiplying both the numerator and denominator by √5:

Cos B = 2√5/5

Now, this equation is not given in options. So, we will find the angle using Sin.

Sin B = AC/BC

Sin B = 3/3√5

Sin B = 1/√5

Now, multiplying both the numerator and denominator by √5:

Sin B = √5/5

Hence, the correct option is B.

To know more about measure of angle here

https://brainly.com/question/27681289

#SPJ1

1.31 (a) possible coverage error: only employees in a specific division of the company were sampled. (b) possible nonresponse error: no attempt is made to contact nonrespondents to urge them to complete the evaluation of job satisfaction. (c) possible sampling error: the sample statistics obtained from the sample will not be equal to the parameters of interest in the population. (d) possible measurement error: ambiguous wording in questions asked on the questionnaire.

Answers

B. No attempt is made to contact noncorrespondents.

What is probability?The area of mathematics known as probability deals with numerical representations of the likelihood that an event will occur or that a statement is true.An event's probability is a number between 0 and 1, where, roughly speaking, 0 denotes the event's impossibility and 1 denotes certainty.The likelihood that an event will occur increases with its probability.A straightforward illustration is tossing a fair (impartial) coin.The chance of both outcomes ("heads" and "tails") is equal because the coin is fair, "heads" is more likely than "tails," there are no other conceivable outcomes, and the likelihood of either outcome is half.

Hence,B. No attempt is made to contact noncorrespondents.

learn more about probability click here:

brainly.com/question/13604758

#SPJ4

8–7. The block brake conit of a pin-connected lever and
friction block at B. The coefficient of tatic friction between
the wheel and the lever i m = 0. 3, and a torque of 5 N # m
i applied to the wheel. Determine if the brake can hold the
wheel tationary when the force applied to the lever i
(a) P = 30 N, (b) P = 70 N

Answers

If the brake can hold the wheel stationary when the force applied to the lever i

(a) P = 30 N     -> No

(b) P = 70 N     -> Yes

To hold lever,

x + Σ[tex]M_{O} = 0[/tex]             [tex]F_{B}(0.15) - 5 = 0[/tex]            [tex]F_{B} = 33.333 N[/tex]

Require

[tex]N_{B} = \frac{33.333}{0.3}[/tex]

      = 111.1 N

Lever,

x + Σ[tex]M_{A} = 0[/tex]            [tex]P_{Reqd.}[/tex](0.6) - 111.1(0.2) - 33.333(0.5) = 0

[tex]P_{Reqd.}[/tex] = 39.8 N

(a) P = 30 N < 39.8 N           No

(b) P = 70 N > 39.8 N           Yes  

To learn more about force here:

https://brainly.com/question/7362815

#SPJ4

l
Ashley's bill for dinner at a restaurant was $100. She left a 15% tip. What was the amount
of the tip?

Answers

Answer:

Step-by-step explanation:

ashleys bill was 100 so by knowing any percentage times 100 is itself ashleys tip was $15

Answer:

the amount on the tip is 15.00$

but when you add the amount of the total cost and the tip it would be $115 dollar they spend

Step-by-step explanation:

Write the equation of the line that passes through the points (1,-3) and (3,3)

Answers

The equation of line is y = 3x - 6

Define Equation of line.

A line in geometry is a collection of points that can be stretched infinitely in two different ways. To put it another way, a line is created by continually extending the two end points in any direction. As a result, we can assert that a line lacks end points. However, because it just has length and no width, it is a one-dimensional geometric form. Based on the information provided, we can create multiple forms of equations for lines algebraically, such as point slope form, slope-intercept form, and two point form. Two Point Form Formula,

          y - y₁ = m (x - x₁)  

We have the given points,

(1, -3) and (3, 3)

Let, (x₁ , y₁) = (1, -3)     and,

(x₂, y₂) = (3, 3)

Formula for equation of line is,

y - y₁ = m (x - x₁)

Where m is slope and for that we have formula for slope is

m =  (y₂ - y₁) / (x₂ - x₁)

plug in the values,

m = (3 - (-3) ) / (3 - 1)

m = 6 / 2

m = 3

Now, we get the slope. Now, just put the values in equation of line formula

y - y₁ = m (x - x₁)

y - (-3) = 3 (x - 1)

y + 3 = 3x - 3

y = 3x - 6

Hence, the equation of line is y = 3x - 6

To read more about Equation of line.

https://brainly.com/question/19417700

#SPJ9

All stereos are 20
%
off of the original price. If stereos normally cost $160, what is the sale price?

Answers

20% of 160

160 divided by 10 = 16

16x2 = 32

160-32= $128

Answer:

128 dollars

Step-by-step explanation:

hope it helped! :)

Write a rate that represents the situation.


Need help on 5 & 6

Answers

Answer:

5.  (6 hours)/(1 day)

6.  (8 quarts)/(1 container)

Step-by-step explanation:

5.  The number line shows both hours and days.  It does not indicate why the relationship exists.  Perhaps the hours are the ones worked in a day.  What’s important is whether the relationship between the two is the same for all data pairs.  E.g., 12/2 = 6; 36/6 = 6; 60/6 - 5, and so forth.  They are all the same ratio, or rate: (6 hours)/(1 day)

6.  The number line shows both quarts and containers.  The  relationship between the two is the same for all data pairs.  E.g., 8/5,  16/10 or 8/5; 24/15 or 8/5; 32/20 or 8/5, and 40/25 or 8/5.   They are all the same ratio, or rate: (8 quarts)/(1 container).

a baseball player averages 2 hits per game. The player already has 18 hits this season. another player has 22 hits so far this season and averages 1 hit per game. After how many games will both players have the same number of hits?

Answers

Answer: 20

Step-by-step explanation:

i got 100

How does a domain restriction placed on a non-invertible function affect its inverse? drag a function or an interval into each box to correctly complete the statement.

Answers

A domain restriction placed on a non-invertible function affect its inverse.

Using inverse function concepts, it is found that:

The inverse of the function is [tex]y = \sqrt{x+3}-1[/tex], and the domain of the inverse function is [tex]\left[-3,\infty ][/tex]

   A function will have an inverse if for each output, there is only one respective input.

   The domain of the inverse is the range of the original function.

The function given is:

[tex]f(x) = (x+1)^{2} -3[/tex]

   It's range is [tex]\left[-3, \infty][/tex], which will be the domain of the inverse.

To find the inverse, we exchange x and y, and isolate y, then:

[tex]y = (x+1)^{2} - 3\\ \\x = (y+1)^{2} - 3\\\\(y+1)^{2} = x+3\\ \\\sqrt{((y+1)^{2}} = \sqrt{x+3} \\\\y+1 = \sqrt{x+3}\\\\y = \sqrt{x+3} -1[/tex]

The inverse of the function is [tex]y = \sqrt{x+3} -1[/tex], and the domain of the inverse function is [tex]\left[-3, \infty][/tex].

To learn more about invertible function click here https://brainly.com/question/3831584

#SPJ4

Probability: Discrete, Binomial, Normal
IB Math Applications SL
1. Jae Hee plays a game involving a
biased six-sided die. The score for
the game, X, is the number which
lands face up after the die is rolled.
The table shows the probability
distribution for X.
a. Find the exact value of p.
Score x
P(X=x)
Name: Somatar
Date:
-3
1
18
-1
P
Period:
0
3
18
1 point: Correct p-value
b. Jae Hee plays the game once. Calculate the expected score.
1
1
18
1 point: Correct substitution into formula
2 points: Correct substitution and answer
c. Jae Hee plays the game twice and adds the two scores together.
Find the probability Jae Hee has a total score of -3.
2
18

Answers

The precise value of p is 4/18. Jae Hee's expected score in the game is 1.833. Jae Hee has a 1/162 chance of receiving a total score of -3. The likelihood that something will happen is defined as probability.

How to calculate Probability ?Probability is calculated by dividing the number of possible outcomes by the total number of outcomes. Probability and odds are distinct concepts. Odds are the probability that something happens divided by the probability that it does not happen. Probability is a branch of mathematics that deals with numerical representations of the likelihood of an event occurring or a statement being true.

Therefore,

Value of p is,

∈p = 1

1/18 + p + 3/18 + 1/18 + 2/18 + 7/18 = 1

14/18 + p = 1

p = 4 / 18

So the exact value of p is 4/18

Expected score of the game :

∈(x) = ∈xi pi

= -3 (1/18) - 1 (4/18) + 0 (3/18) + 1 (1/18) + 2 (2/18) + 5 (7/18)

= 33/18

= 1.833

Expected score of the game Jae Hee plays is 1.833

Calculate Probability of Jae Hee getting total score of -3

p (total score -3 )

= p(x= -3) and p(x=0) + p(x=0) and p (x=3)

= 1/18 × 1/18 + 1/18 × 1/18

=1/162

Probability of Jae Hee getting total score of -3 is 1/162

To learn more about Probability, refer to:

brainly.com/question/24756209

#SPJ13

Answer:

Step-by-step explanation:

a month with 31 days has the same number of mondays and wednesdays.how many of the seven days of the week could be the first day of this month?

Answers

Answer:

sunday and saturday i believe

Step-by-step explanation:

lmk if it helps! :)

a team of three high jumpers all have a personal record that is at least 6 feet and less than 7 feet. is it necessarily true that two of the team members must have personal records that are within four inches of each other? what if there are four jumpers? heights are measured to within a precision of 1/4 inch.

Answers

The statement that two of the team members must have personal records that are within four inches of each other it is not true for three high jumpers, and true for four high jumpers.

1 foot =  12 inches

So, for three high jumpers the statement is not true. A counterexample,

First player = 6 feet

Second player = 6 feet 6 inches

Third player = 6 feet 11 inches

Thus, personal record each player have interval more than 4 inches of each other.

For four high jumpers the statement is true. Since there are 4 jumpers and 3 intervals by the pigeonhole principle, at least two of the four jumpers have intervals within four inches. For example, in the image, the new jumpers should be placed in areas A, B, C and this makes at least two of the four jumpers have an interval of four inches.

Thus, the statement is not true for three jumpers and true for four jumpers.

Learn more about pigeonhole principle here:

brainly.com/question/29242775

#SPJ4

Which system of linear inequalities is graphed?

A.
x<-3
y ≤-x+1

B.
x<-3
y ≤-x-1

C.
x<-2
y ≤-x-1

D.
x ≤-3
y ≤-x-3

Answers

Answer:

A x<-3

y ≤-x+1

Step-by-step explanation:

x<-3

y ≤-x+1

The function c(x) = 14x + 13 represents the cost (in dollars) of renting a surfboard, where x is the number of hours you rent
the surfboard.
a. Graph the function and identify its domain and range.

Answers

The domain of this function c(x) = 14x + 13 using interval notation is (0, ∞).

The range of this function c(x) = 14x + 13 using interval notation is (13, ∞).

How to identify the domain and range any graph?

In Mathematics, the horizontal portion of any graph is used to represent all domain values and they are both read and written from smaller to larger numerical values, which simply means from the left of any graph to the right.

Additionally, the vertical extent of any graph represents all its range values and they are always read and written from smaller to larger numerical values, and from the bottom of a graph to the top.

By critically observing the graph of this function shown in the image attached below, we can reasonably infer and logically deduce the following:

Domain = (0, ∞).

Range = (13, ∞).

Read more on domain here: brainly.com/question/17196520

#SPJ1

Given the explicit formula u(n) = 4n - 1, where u(1) = 1, find the recursive formula for u(n).
A. u(1) = 1 and u(n + 1) = 4u(n), for n = 1, 2, 3, . . .
B. u(1) = 1 and u(n + 1) = u(n) + 4, for n = 1, 2, 3, . . .
C. u(1) = 1 and u(n + 1) = u(n) + 8, for n = 1, 2, 3, . . .
D. u(1) = 1 and u(n + 1) = 8u(n), for n = 1, 2, 3, . . .

Answers

The Recursive formula for the given explicit formula is u(1) = 1 and u(n+1) = u(n) + 4 , for n = 1 , 2 , 3 ....  , the correct option is (b) .

In the question ,

it is given that ,

the explicit formula is u(n) = 4n - 1 .

and u(n) = 1 ,

we need to find the recursive formula ,

We substitute n + 1 in place of n , in the equation u(n) = 4n - 1

after substitution , we get

u(n+1) = 4(n + 1) - 1

u(n+1) = 4n + 4 - 1

u(n+1) = 4n + 3

Subtracting u(n) = 4n - 1 from u(n+1) = 4n + 3  ,

we get

u(n+1) - u(n) = 4n + 3 - (4n - 1)

u(n+1) - u(n) = 4n + 3 - 4n + 1

u(n+1) - u(n) = 3 + 1

u(n+1) - u(n) = 4

u(n+1) = u(n) + 4

Therefore , The Recursive formula for the given explicit formula is u(1) = 1 and u(n+1) = u(n) + 4 , for n = 1 , 2 , 3 ....

Learn more about Recursive Formula here

https://brainly.com/question/1827481

#SPJ1

Find the slope of each line.

11. through (3, 5), parallel to y = 5x-1

12. through (-0.5, 0.5), perpendicular to y = -2x - 4

Answers

The slope is 5. The slope of a line parallel to this equation will be 5, regardless of its position.

To find the line that goes through the point (3,5) that has slope = 5.

y = 5x + b

now use the point to find 'b'

5 = 5(3) + b

b = -10

perpendicular: [tex]\frac{1}{2}[/tex]

[tex]y-0.5= \frac{1}{2} \times(x + 0.5)[/tex]

[tex]y - 0.5 = \frac{1}{2}x + 0.25[/tex]

[tex]y = \frac{1}{2}x+\frac{3}{4}[/tex]

What is Slope?

The slope of a line is a measure of its steepness. Mathematically, slope is calculated as "rise over run" (change in y divided by change in x).

Find out more about Slope of line:

https://brainly.com/question/3493733

#SPJ13

Mary places a 15-foot ladder against her house. The base of the ladder is 3 feet from her house.
How high up, to the nearest tenth of a foot, does the ladder reach on Mary's house?

A. 14.7
B. 14.8
C. 14.9
D. 15.3

Answers

Answer:

14.7

Step-by-step explanation:

hopes this helps please mark brainliest

A group of 60 people are going on a trip to an amusement park that has dollar coasters and water rides. The number of people who like roller coasters is 42, the number who do not like water rides is 23 and the number who do not like either kind of ride is 8. Determine which Venn diagram accurately represents the people going on the trip.

Answers

The no. of people whole likes only roller coasters is 19.

What are sets and subsets?

A set is a collection of well-defined objects.

A subset contains all the elements or a few elements of the given set.

The improper subset is when it contains all the elements of the given set and the proper subset is when it doesn't contain all the elements of the given set.

Let N(U) be the no. who likes at least one of the rides which is

N(U) = (60 - 8) = 52 as 8 people do not like either kind of ride.

Given, The number of people who like roller coasters is 42 and the number who do not like water rides is 23.

∴ The no. of people who only likes roller coasters is (42 - 23) = 19.

learn more about sets and Venn diagrams here :

https://brainly.com/question/28318748

#SPJ1

6. A sunflower was 12 inches when first planted. Since then it has grown 34 of an inch each month.
A. Write a linear equation.
B. Identify the slope and y-intercept.
C. How tall was it after a year?

Answers

The linear equation will be y=34x+12 and the slop is 34 and y intercept is 12. And the sunflower will be 420 inches tall after a year.

Here, the sunflower was 12 inches when first planted and it grows 34 of an inch each month

So, the linear equation will be

y=34x+12

Where,

y=height

x=month

now,

the slope-intercept equation is

y=mx+c

Where,

m=slope

c=y intercept

Comparing this equation with equation (1)

We will get

m=34

c=12

so, the slope is 34 and y intercept is 12.

now,

The tree is 12 inches when it first planted and since then it grows 34 of an inch every month

So,

Height of tree after a year =12+34×12

Height of tree after a year = 12+408

Height of tree after a year = 420 inches.

Hence,

The tree will be 420 inches tall after a year.

To know more about linear equation refer to :

https://brainly.com/question/12788590

#SPJ9

reflect shape A in the x-axis

Answers

Answer:

points: (4,-1) (1,-1) (4,-3)

Step-by-step explanation:

salma has set up a lemonade stand outside her house and sells small cups and large cups of lemonade. each small cup holds 12 ounces of lemonade and each large cup hold 16 ounces of lemonade. salma sold 60 cups of lemonade totaling 920 ounces. determine the number of small cups sold and the number of large cups sold

Answers

If Salma sold 60 cups of lemonade with total 920 ounces, then the number of small cups is 10 and the number of large cups is 50.

The problem can be translated into a system of linear equations.

Let:

p = the number of small cups sold

q = the number of large cups sold

"Salma sold 60 cups"  can be translated into equation:

p + q = 60   (equation 1)

"totaling 920 ounces" can be translated into equation:

12p + 16q = 920  (equation 2)

Multiply equation 1 by 12 then substracted from equation 2:

12p + 16q = 920

12p + 12q = 720 _

          4q = 200

            q = 50

Substitute q = 50 into equation 1:

p + 50 = 60

p = 10

Hence, the number of small cups = 10, the number of large cups = 50.

Learn more about equations here:

https://brainly.com/question/13729904

#SPJ4

help meeeeeeeeeeee pleaseee rnnnnn!!!!!!!!!!!!!!!!!!!!!!!!!!!!!!!!!!!!!!!!!!!!!!!!!!!!!!!!!!!!!!!!!!!!!!!!!!!!!help meeeeeeeeeeee pleaseee rnnnnn!!!!!!!!!!!!!!!!!!!!!!!!!!!!!!!!!!!!!!!!!!!!!!!!!!!!!!!!!!!!!!!!!!!!!!!!!!!!!

Answers

The completed table of coordinates for the exponential function can be  presented as follows;

x; -2,       -1,  0, 1,       2

y ; 16/9, 4/3, 1, 3/4, 9/16

Please find attached the graph of the function created with MS Excel

The correct graph is therefore, option D

What is an exponential function?

An exponential function is one that has the argumant as an exponent

The values in the table can be found as follows;

[tex]f(x) = \left(\frac{3}{4} \right)^x[/tex]

[tex]f(x) = \left(\frac{3}{4} \right)^{-2} = \dfrac{16}{9}[/tex]

[tex]f(x) = \left(\frac{3}{4} \right)^{-1} = \dfrac{4}{3}[/tex]

[tex]f(x) = \left(\frac{3}{4} \right)^{0} = 1[/tex] The y-intercept

[tex]f(x) = \left(\frac{3}{4} \right)^{1} = \dfrac{3}{4}[/tex]

[tex]f(x) = \left(\frac{3}{4} \right)^{2} = \dfrac{9}{16}[/tex]

The completed table is therefore;

x; -2,       -1,  0, 1,       2

y ; 16/9, 4/3, 1, 3/4, 9/16

The graph of the function ,  [tex]f(x) = \left(\frac{3}{4} \right)^x[/tex], therefore, has a y-intercept of 1, and a maximum value of 16/9 at x  = -2

The best cprect option  for the graph is therefore option  D

Learn more about exponential functions here:

https://brainly.com/question/11464095

#SPJ1

I’ll give brainlist!!!
What is the slope?

Answers

The slope of the line is 1/3

First we have to choose two points from the graph

The first point = (1, -5)

The second point = (7, -3)

The slope of the line is the change is y coordinate with respect to the change in x coordinate of the function

The slope of the line m = [tex]\frac{y_2-y_1}{x_2-x_1}[/tex]

Where m is the slope of the line

[tex](x_1,y_1)[/tex] is the coordinate of the first point

[tex](x_2,y_2)[/tex] is the coordinate of the second point

Substitute the values in the equation

The slope of the line = (-3 - (-5) / (7 - 1)

= (-3+5) / 6

= 2 / 6

= 1/3

Hence, the slope of the line is 1/3

Learn more about slope of the line here

brainly.com/question/16180119

#SPJ1

Other Questions
you are troubleshooting a friend's computer and your diagnostic software reports a problem with the mft. what file system is your friend's computer using? Calculate70cos 20rounded to 1 d.p.70cos 20 is a faction btw Geography medc/ledcs export raw materials Pls list step by step : calculus : Use a linear approximation to estimate the value of 33. Express your answer as an exact rational number screening for this most common birth defect is required by law in most states. each nurse should know the law for his or her state and the requirements for screening. the nurse would expect a newborn to be screened for which defect as the most common? in an attempt to understand the mechanisms underlying consumption, researchers have drawn from many areas such as psychology, sociology, and marketing. the result is the social science known a Every month, the bank withdraws $15 from Betsy's checking account as a service fee. Betsy has budgeted $75 for the next few service fees. For how many months will the service fee be covered? A line with a slope of 4/7 passes through the points (h,10) and (4,6). What is the value of h? a company currently sells 15,000 units a month for $50 each, has variable costs of $20 per unit, and fixed costs of $300,000. the company is considering increasing the price of its units to $60 per unit. if the price is changed, how many units will the company need to sell for profit to remain the same as before the price change? TRANSLATING and SOLVING EquationsThe product of a number and -7 is 63Equation: PLEASE. HELP! I WILL NAME YOU THE BRAINLIEST!!!!!!!!!!If a student chooses not to finish the bachillerato, what choices are there in many Spanish-speaking countries to prepare him or her for the workforce? [Choose all that apply].pre-university program.entering the workforce itself.sports program.vocational school A piece of what nasa space shuttle was recently discovered by a film crew in the atlantic ocean?. write a function called tokenizetelnum that inputs a telephone number as a string in the form (555) 555-5555. the function should use function strtok to extract the area code as a token, the first three digits of the phone number as a token and the last four digits of the phone number as a token. the seven digits of the phone number should be concatenated into one string. the function should convert the area-code string to int and convert the phone-number string to long. both the area code and the phone number should be printed. Im tryin to get ready for the test 11 times the product of 19 and a number g? In parallelogram EFGH, the measure of angle G is (4x + 16) and the measure of angle H is (2x - 16). What is the measure of angle H?324488136 A character who acts in a effrontery manner in the crucible ? PLEASE HELP 34 pointsssss Two construction workers are attempting to pull a slab of concrete off of the ground. Each of the workers are pulling up with a force of 225 n, while the slab of concrete applies a gravitational force of 400 n in the opposite direction, remaining stationary. If the pulling forces applied to the slab of concrete are assigned a positive value, what is the net force of the scenario, and in which direction will the slab of concrete move?. One objective of the immigration act of 1924 was to reduce the number of immigrants entering the united states each year. The legislation reflects a pattern of intolerance that was present in the 1920s. Which other development of the 1920s also reflects this pattern of intolerance?.